Put the following equation of a line into slope-intercept form, simplifying all fractions.
4x-20y=-160

Answers

Answer 1

The equation of the line 4x-20y = -160 in the slope intercept form is y = x/5 + 8 .

The equation of the line in slope intercept form is written as

y = mx + c ,

where m is the slope of the line , and c is the y intercept .

In the question ,

the equation of the line is given

equation of line is given as 4x-20y = -160

to convert the equation in slope intercept form , we rewrite it as

20y = 4x+160

dividing both sides by 20 ,

we get

20y/20 = 4x/20 + 160/20

y = x/5 + 8

hence ,the slope intercept form is y = x/5 + 8 , where 8 is the y intercept and 1/5 is the slope .

Therefore , the equation of the line 4x-20y = -160 in the slope intercept form is y = x/5 + 8 .

Learn more about Lines here

https://brainly.com/question/21206550

#SPJ1


Related Questions

Find the equation for the circle with a diameter whose endpoints are (8,17) and (4, - 15).

Answers

Answer:

Given that,

Circle with a diameter whose endpoints are (8,17) and (4, - 15).

To find the equation for the circle

we know that,

Equation of the circle is of the form,

[tex](x-h)^2+(y-k)^2=r^2[/tex]

where r is the radius and (h,k) is the center of the circle.

If the end points of the diameter is given, then the equation of the circle is,

[tex](x-x1)(x-x2)+(y-y1)(y-y2)=0[/tex]

Substitute the points we get,

[tex](x-8)(x-4)+(y-17)(y+15)=0[/tex]

On simplifying this we get,

[tex]x^2-8x-4x+32+y^2-17y+15y-255=0[/tex][tex]x^2+y^2-12x-2y-223=0[/tex]

URGENT!! ILL GIVE
BRAINLIEST!!!!! AND 100
POINTS!!!!!

Answers

The correct option b: 49° because the angles are supplementary to angle d.

What is defined as supplementary angles?The definition of supplementary is linked to angles that form a straight angle when joined together. When two angles add up to 180 degrees, they are referred to as supplementary angles. If two angles are supplementary. One of its angles is just an acute angle, while another is an obtuse angle.

For the given question.

∠d = 131 degrees.

∠d + ∠f = 180  (supplementary angle)

∠f = 180 - 131

∠f = 49 degrees

Now,

∠f = ∠g = 49 degrees (vertically opposite angles)

∠f = ∠c = 49 degrees (alternate interior angles)

Thus, the angle d is supplementary to angle f, c and g.

To know more about the supplementary angles, here

https://brainly.com/question/12919120

#SPJ13

i need help with this problem

Answers

Answer:

whats the problme?

Step-by-step explanation:

Given the formula FV = P + Prt, what is the future value of a savings account that had an initial deposit of $7,900 earning 6.5% simple interest for 4 years?$10,074.23$9,954.00$9,376.85$10.756.43None of these choices are correct.

Answers

Step 1: Use the formula below to find the future value:

Fv = P + Prt

P = present value or initial deposit

r = rate in %

t = time in years

Step 2: List the given data

P = $7900

r = 6.5% = 0.065

t = 4 years

Step 3: Substitute the values of P, r and t to find the future value.

FV = P + Prt

= 7900 + 7900 x 0.065 x 4

= 7900 + 2054

= $9954.00

Stop 4: Final answer

Future value = $9954.00

What is the slope of a line that passes the points (-1,4 ) and ( 3,9 ) ?

Answers

Answer:

slope = 5/4

Step-by-step explanation:

What is the slope of a line that passes the points (-1,4 ) and ( 3,9 ) ?

slope = change in y ÷ change in x

slope = (9-4) ÷ (3 - (-1))

slope = 5/4

Use the ALEKS calculator to write as a percentage.
31
32
Round your answer to the nearest tenth of a percent.
0%
X 5
?

Answers

Divide 31 into 32 and then multiply the result by 100 to get the percent:

[tex]\begin{gathered} \frac{31}{32}=0.96875 \\ \\ 0.96875*100=96.875 \end{gathered}[/tex]Then, to the nearest tenth of a percent the given fraction is 96.9%

1)EF=JHFEG = HJGXProve FEG = HJGStatement1) EF = JHReason1) given2)2) given3) Vertical angles are conguent3)4)4)Theorem

Answers

2.

[tex]\angle FEG\cong\angle HJG[/tex]

3.

[tex]\angle HGJ\cong\angle EGF[/tex]

10. The product of 6 and a number when added to 5 is equal to 1 less than 9 times the number.

Answers

6x+5=9x-1
6x+4=9x
3x=4
X=4/3

Answer:

x = 2

Step-by-step explanation:

Hello!

Let the unknown number be x. The product of 6 and that number can be represented by 6x. And since we are adding 5, we can use 6x + 5 to represent that expression.

This equation is equal to the difference of 9x and 1, or 9x - 1.

Solve for x6x + 5 = 9x - 15 = 3x - 16 = 2x2 = x

The value of x is 2.

The total cost, in dollars of a membership in a fitness center is given by the function c(m) = 40m +10, where m is the number of months a person is a member. In dollars, how much is the cost of a membership for 1 year?

Answers

The cost of a membership for 1 year is 490

How to determine the cost of a membership for 1 year?

The equation of the membership is given as

c(m) = 40m + 10

From the question, we understand that:

m represents the number of months

For the cost of a membership for 1 year, the number of months is 12

i.e. m = 12

Substitute the known values in the above equation

So, we have

c(12) = 40 * 12 + 10

Evaluate

c(12) = 490

Hence, the cost is 490

Read more about linear equation at

https://brainly.com/question/4074386

#SPJ1

Given the general form: F( x )= a(x^2)+bx+cConvert it to vertex form (also known as standard form) by putting the values for a,h and k into the correct boxes.F(x)=a(x-h)^2+kIdentify the vertex(x,y)General form: F( x )=1 x^2+6 x +-1 Vertex form: F( x )= Answer for part 1 and coordinate 1 (x- Answer for part 1 and coordinate 2 )^2 +Answer for part 1 and coordinate 3Vertex: (Answer for part 2 and coordinate 1,Answer for part 2 and coordinate 2)

Answers

[tex]\begin{gathered} Vertex\colon\: (-3,-10) \\ Vertex\: form\: equation\colon\: f(x)=(x+3)^2-10 \end{gathered}[/tex]

1) In order to convert from the standard version to the vertex form we'll need to find the vertex of that parabola:

[tex]f(x)=x^2+6x-1[/tex]

We can find the vertex, using these formulas:

[tex]\begin{gathered} x=h=-\frac{b}{2a}=\frac{-6}{2}=-3 \\ k=(-3)^2+6(-3)-1=9-18-1=9-19=-10 \\ V(-3,-10) \end{gathered}[/tex]

So this is the vertex of that parabola at point (-3,-10)

2) Now, note that the coefficient a = 1, and with the vertex, we can now rewrite that equation into the vertex form:

[tex]\begin{gathered} y=a(x-h)^2+k \\ y=(x-(-3))^2+(-10) \\ y=(x+3)^2-10 \end{gathered}[/tex]

Select the correct answer. What is the solution to the equation? (x - 2)^1/2 + 4 = x A. -3 and -6 B. 3 and 6 C. -3 D. 6

Answers

Answer:

D.  x = 6

Step-by-step explanation:

Given equation:

[tex](x-2)^{\frac{1}{2}}+4=x[/tex]

Subtract 4 from both sides:

[tex]\implies (x-2)^{\frac{1}{2}}+4-4=x-4[/tex]

[tex]\implies (x-2)^{\frac{1}{2}}=x-4[/tex]

Square both sides:

[tex]\implies \left( (x-2)^{\frac{1}{2}}\right)^2=(x-4)^2[/tex]

[tex]\implies x-2=(x-4)^2[/tex]

Expand the brackets on the right side:

[tex]\implies x-2=(x-4)(x-4)[/tex]

[tex]\implies x-2=x^2-8x+16[/tex]

Subtract x from both sides:

[tex]\implies x-2-x=x^2-8x+16-x[/tex]

[tex]\implies -2=x^2-9x+16[/tex]

Add 2 to both sides:

[tex]\implies -2+2=x^2-9x+16+2[/tex]

[tex]\implies 0=x^2-9x+18[/tex]

[tex]\implies x^2-9x+18=0[/tex]

Factor the left side of the equation:

[tex]\implies x^2-6x-3x+18=0[/tex]

[tex]\implies x(x-6)-3(x-6)=0[/tex]

[tex]\implies (x-3)(x-6)=0[/tex]

Apply the zero-product property:

[tex]\implies x-3=0 \implies x=3[/tex]

[tex]\implies x-6=0\implies x=6[/tex]

Therefore, the solutions of the quadratic equation are:

[tex]x=3, \quad x=6[/tex]

Input both solutions into the original equation to check their validity:

[tex]\begin{aligned}x=3 \implies (3-2)^{\frac{1}{2}}+4&=3\\(1)^{\frac{1}{2}}+4&=3\\1+4&=3\\5&=3\end{aligned}[/tex]

[tex]\begin{aligned}x=6 \implies (6-2)^{\frac{1}{2}}+4&=6\\(4)^{\frac{1}{2}}+4&=6\\2+4&=6\\6&=6\end{aligned}[/tex]

Therefore, the only valid solution to the given equation is x = 6.

Answer:

The answer is D. 6

Step-by-step explanation:

Add both sides:

(x - 2)^1/2 + 4 + (-4) = x + (-4)

(x - 2)^1/2 = -4

Solve exponent:

(x - 2)^1/2 = x - 4

((x - 2)^1/2)^2 = (x - 4)^2

x − 2 = x^2 − 8x + 16

x − 2 − (x^2 − 8x + 16) = x^2 − 8x + 16 − (x^2 − 8x + 16)

−x^2 + 9x − 18 = 0

(−x + 3)(x − 6) = 0

−x + 3 = 0 or x − 6 = 0

x = 3 or x = 6

Check the answers: (Plug them in to see what will work.)

x = 3 (won't work)

x = 6 (does work)

Therefore,

x = 6

While solving an equation 2x^2+32=0 the answer calculated is x=+-4i I understand the +- means the answer can be positive or negative but what does the i mean

Answers

Answer:

Explanation:

Given:

[tex]\begin{gathered} 2x^2+32=0 \\ \text{The answer is x=}+-4i \end{gathered}[/tex]

To fully understand how we get the given answer, we simplify the equation first:

[tex]\begin{gathered} 2x^2+32=0 \\ \text{Simplify and rearrange} \\ 2x^2=-32 \\ x^2=-\frac{32}{2} \\ x^2=-16 \\ \end{gathered}[/tex]

Next, we apply the rule:

[tex]\begin{gathered} \text{For x}^2=f(a),\text{ the solutions are } \\ x=\sqrt[]{f(a)} \\ x=-\sqrt[]{f(a)} \end{gathered}[/tex]

So,

[tex]\begin{gathered} x^2=-16 \\ x=\sqrt[]{-16},x=-\sqrt[]{-16} \end{gathered}[/tex]

Then, we also apply the radical rule:

[tex]\begin{gathered} \sqrt[]{-a}=\sqrt[]{-1}\sqrt[]{a} \\ So, \\ x=\sqrt[]{-16} \\ =\sqrt[]{-1}\sqrt[]{16} \\ \text{Then, apply the imaginary number rule:} \\ \sqrt[]{-1}=i \\ \text{Hence,} \\ x=4i \end{gathered}[/tex]

For

[tex]\begin{gathered} x=-\sqrt[]{-16} \\ Use\text{ the same steps} \\ x=-4i \end{gathered}[/tex]

Therefore the x-values are: x=4i, x=-4i. The i on the answer means imaginary number. It is a number that, when squared, has a negative result.

Graph the line that has an z-intercept of (-3,0) and a y-intercept of (0, - 5). What is the slope of this line?

Answers

Answer:

the slope is -5/3

Step-by-step explanation:

it is that because to find slope u do y2-y1/x2-x1

Identifying the Type of Series
3+6+12+ 24 + ...

5+7+10+14+

1+2+3+4+

2+4+2+4+

Answers

1) So look closely at first, you’ll see that each successive term is the twice of preceding term
Thus we get 48 and then 96 and so on
2) we add 2 to the first term to get 7 then we add 3 to get 10 and then 4 to get 14
Thus now we add 5 to get 19 and so on
3) this question speaks for itself
4) it’s just repeating with 2 4 2 4 2 4 and so on

Answer:

geometric, neither, arithmetic, and neither

Step-by-step explanation:

right on edg 2023

8. Find the slope between (-5, 4) & (0,3). * O m = 1/5 O O m = -5 m = 5 O m = -1/5

Answers

We can determine the slope using the following expression:

[tex]m=\frac{y_2-y_1}{x_2-x_1}[/tex]

Now, using the two points given, we have:

[tex]m=\frac{3-4}{0-(-5)}\Rightarrow m=-\frac{1}{5}[/tex]

From this, we have that the slope(m) equals -1/5.

while digging in his garden , will pushes a shovel into the ground at an 80 degree angle with 585 newtons of force . show the resolution of the force into its retangular components

Answers

Solution

Part a

Part b

For this case we can do this:

Fx= 585 N* cos 80= 101.58 N

Fy= 585N * sin 80= 576.11 N

Then the best answer is:

B. (102, 576) N

The tables of ordered pairs represent some points on the graphs of lines q and v. Line 9 Line v Х -9 -3 2. Х 0 10 у 0 18 33 у 10 8 3 Which system of equations is represented by lines q and v? F 21x - y = 9 5x + 6y = 40 G 3x - y = -27 x + 2y = 16 H 21x - y = 9 5x - 6y = 20 3 9x - y = -27 x + 2y = 8

Answers

One of the forms we can write the equation of a line is like this:

y - y1 = m(x - x1)

Where (x1, y1) is a point where the line passes through, and the value of m, the slope of a line is given by the following formula:

[tex]m=\frac{y2-y1}{x2-x1}[/tex]

Then, for the first line (line q), we can take the points (-9, 0) and (-3, 18), then we get:

[tex]mq=\frac{18-0}{-3-(-9)}=\frac{18}{-3+9}=\frac{18}{6}=3[/tex]

By replacing the value of the slope and the coordinates of the point (-9, 0), we get:

y - 0 = 3(x - (-9))

y = 3(x + 9)

y = 3x + 27

y - y = 3x + 27 - y

0 = 3x + 27 - y

-27 = 3x + 27 - 27 - y

-27 = 3x - y

For line v, we can take the points (-4, 10) and (0, 8), then we get:

[tex]mv=\frac{8-10}{0-(-4)}=\frac{-2}{4}=-\frac{1}{2}[/tex]

By taking -1/2 for the slope and the coordinates of the point (0,8), we gat:

y - 0 = -1/2(x - 8)

y = -1/2x + 4, multiplying both sides by 2:

2y = -x + 16

2y + x = -x + x + 16

2y + x = 16

Then, the system represented by the lines q and v is option G

Working 5 hours a day, A can Complete a work in 8 days and working 6 hours a day, B can complete the same work in 10 days. Working 8 hours a day, they can jointly complete the work in how many days?​

Answers

After solving the equation, If both A and B of them worked together, then Working 8 hours a day, they can jointly complete the work in 6 days.

What is an equation?

Two mathematical expressions' values are said to be equal in an equation, which is a statement of this fact. A mathematical formula declares that two things are equal.

The equals sign ('=') is used to indicate it.

Let the work completed be W

For A

W = 5hours = 1/8 days

1 hour =  1/8 days ÷ 5

1 hour = 1/40 days

For B

W = 6 hours = 1/10 days

1 hour = 1/60 days

Add both the equation

1 hours + 1 hours = 1/40 days + 1/60 days

2 hours = 5/120 days

2 hours = 1/24days

If both of them worked for 1 hour a day

1 hour = 1/48 days

If both of them worked for 8hour a day

8 hours = 1/48 × 8

             = 1/6 days

Thus, if both of them worked together, then Working 8 hours a day, they can jointly complete the work in 6 days.

Learn more about equations

https://brainly.com/question/2972832

#SPJ9

Solve the following. List all possible possible solutions for the ambiguous case. #7

Answers

The sum of the interior angles of any triangle is always 180º:

[tex]A+B+C=180[/tex]

Use the equation above and the given data to find C:

[tex]\begin{gathered} C=180º-A-B \\ C=180º-38º-72º \\ C=70º \end{gathered}[/tex]

Law of sines:

[tex]\frac{a}{sinA}=\frac{b}{sinB}=\frac{c}{sinC}[/tex]

Use the pair of ratios for a and b to solve a:

[tex]\begin{gathered} \frac{a}{sinA}=\frac{b}{sinB} \\ \\ a=sinA*\frac{b}{sinB} \\ \\ a=sin38º*\frac{12}{sin72º} \\ \\ a=7.8 \end{gathered}[/tex]

Use the pair of ratios for b and c to solve c:

[tex]\begin{gathered} \frac{c}{sinC}=\frac{b}{sinB} \\ \\ c=sinC*\frac{b}{sinB} \\ \\ c=sin70º*\frac{12}{sin72º} \\ \\ c=11.9 \end{gathered}[/tex]Thenm, the solution for the given triangle is:A=38ºB=72ºC=70ºa=7.8b=12c=11.9

In how many ways can a committee of 5 be chosen from 9 people given that Jones must be one of them?

Answers

There are 126 ways to choose a committee of five from a group of nine people using combinations; one of them must include JONES.

What do we mean by COMBINATIONS?Combinations are a mathematical method for calculating the number of alternative arrangements. In a collection of objects where the order of the selection is irrelevant. You are free to choose any combination of the available things.Mathematically it can be expressed as : nCr = n! / r!(n-r)!

So, we have a comiitte of 5 be chosen from 9 people and JONES must be one of them -

Using the rule of combinantion - nCr = n! / r!(n-r)!We have n = 9 and r= 59C5 = 9! / 5!(9-5)!C(9,5) = 3024/24C(9,5) = 126

Therefore, there are a total of 126 ways in which we can decide that JONES must be on it.

Know more about combinations here:

https://brainly.com/question/28065038

#SPJ9

I'll send a picture! please answer fast!

Answers

Given data:

The given expression for the points is,

[tex]10w-3t+5[/tex]

Thus, the expression or

I need help with this practice,I will send you an additional pic that goes along with this problem Freya went to her local park to find 5 organisms/species She found 5 and wrote down the name of these organisms and the quantity of each she seen:Eastern gray squirrels/14 individualsWolf spiders/2 individualsPaper wasps/9 individualsBlack vulture/1 individualNorthern cardinals/7 individuals

Answers

ANSWER :

The answer is 5/33 or 0.15

EXPLANATION :

From the problem, we have a total of 5 number of species.

Using the given Biodiversity Index formula :

[tex]\frac{\text{ total number of species}}{\text{ total number of individuals}}=\frac{5}{14+2+9+1+7}=\frac{5}{33}\quad or\quad0.1515[/tex]

What is the value of (–3 + 3i) + (–2 + 3i)?

Answers

Answer: -5+6i

Step-by-step explanation:

The figure shows a quarter circle and an equilateral triangle. What is thearea of the shaded part? Give your answer to 3 significant figures. (Take it= 3.14.)7 cm

Answers

Since the triangle is equilateral, all of its interior angles have a measure of 60º.

Substract the area of the triangle from the area of a circular sector with radius 7cm enclosed by an angle of 60º to find the area of the shaded region.

The area of an equilateral triangle with side length L is:

[tex]A=\frac{\sqrt[]{3}}{4}L^2[/tex]

The area of a circular sector of radius r enclosed by an angle of θ degrees is:

[tex]A=\frac{\theta}{360}\times\pi r^2[/tex]

Replace θ=60 and r=7cm to find the area of the circular sector:

[tex]A_c=\frac{60}{360}\times3.14\times(7\operatorname{cm})^2=25.643\ldots cm^2[/tex]

Replace L=7cm to find the area of the triangle:

[tex]A_T=\frac{\sqrt[]{3}}{4}\times(7\operatorname{cm})^2=21.2176\ldots cm^2[/tex]

Then, the area of the shaded region is:

[tex]\begin{gathered} A_C-A_T=25.6433\ldots cm^2-21.2176\ldots cm^2 \\ =4.4257\ldots cm^2 \\ \approx4.43\operatorname{cm}^2 \end{gathered}[/tex]

Therefore, the area of the shaded region to 3 significant figures, is:

[tex]4.43\operatorname{cm}^2[/tex]

Jamar finds some nickels and quarters in his change purse. How much money (in cents) does he have if he has 4 nickels and 11 quarters? How much money (in cents) does he have if he has n nickels and q quarters?

Answers

1 nickel is worth 5 cents and 1 quarter is worth 25 cents.

So, if he has 4 nickels, this is worth 4 times 5 cents and if he has 11 quarters, this is worth 11 times 25 cents.

Both are worth the sum of these, so:

[tex]4\cdot5+11\cdot25=20+275=295[/tex]

So, he has 295 cents.

If he has n nickels and q quarters, he has 5 times n plus 25 times q worth, so he has:

[tex]5n+25q[/tex]

use the two given points and calculate the slope.(6,4),(4,-1)

Answers

EXPLANATION:

Given;

We are given two points which are shown below;

[tex]\begin{gathered} (6,4) \\ (4,-1) \end{gathered}[/tex]

Required;

We are required to calculate the slope.

Step-by-step solution;

To calculate the slope given two points, we shall use the following formula;

[tex]\begin{gathered} Slope: \\ m=\frac{y_2-y_1}{x_2-x_1} \end{gathered}[/tex]

Where the variables are;

[tex]\begin{gathered} (x_1,y_1)=(6,4) \\ (x_2,y_2)=(4,-1) \end{gathered}[/tex]

We can now substitute these values and solve;

[tex]m=\frac{-1-4}{4-6}[/tex][tex]m=\frac{-5}{-2}[/tex][tex]m=\frac{5}{2}[/tex]

Therefore.

ANSWER:

[tex]m=\frac{5}{2}[/tex]

What is Qualitative Data and what is the discreate and Continuous in qualitative data?

Answers

Answer:

Qualitative data is the data that is not represented by numbers, for example, favorite food or country.

On the other hand, the quantitative data is represented by numbers and it is classified as discrete and continuous. The discrete data is the data that only can take specific values, for example, the number of people is always a whole number, there can't be 5.5 people. The continuous data is the data that can take decimal values, for example, the mass of an object can be 4.06 kg.

Travelers arriving at Cape Town International Airport 70% of the travelers fly on major airlines, 20% by on privately owned planes and the remainder by on commercially owned planes not belonging to a major airline. Of those traveling on major airlines, 40 degrees are traveling for business reasons, whereas 70% of those arriving on private planes and 80% of those arriving on other commercially owned planes are traveling for business reasons Suppose that we randomly select one person arriving at this airport. What is the probability that the traveler is flying privately for business reasons?

Answers

Step 1

Given;

[tex]\begin{gathered} 20\text{\% fly privately owned jets} \\ 70\text{\% of those arriving on the plane are travelling for business reason} \end{gathered}[/tex]

What’s the correct answer answer asap for brainlist

Answers

Answer:

A. a chorus

Step-by-step explanation:

If sin 0= -3/5 in quadrant 3, what is cos 0?

Answers

Given

[tex]sin\theta=-\frac{3}{4}[/tex]

Solution

Recall : SOHCAHTOA

The final answer

Option A

[tex]Cos\text{ }\theta=-\frac{4}{5}[/tex]

Other Questions
Acoording to the school survery, 20% of the students at rockwood junior high school speak spanish. There are 25 students at the school who speak spanish. How many students were surveyed? is nh4hco3 acidic, neutral, or basic? if is it not neutral, compare their ka and kb values to determine if it is acidic or basic and explain your reasoning #3 TV Aspect RatioUsing your knowledge of aspect ratios, find the difference between the actualscreen size or area of an older style TV with an aspect ratio of 4:3 and a newer TVwith an aspect ratio of 16:9. Assume that both TV's are 52 inches in height. Whatis the difference in the areas of the two TV's? (show all work and justify youranswer) (8 points) Two ice skaters stand facing each other at rest on a frozen pond. They push off against one another and the 49 kg skater acquires a speed of 2.10 m/s. If the other skater acquires a speed of 3.81 m/s, what is her mass in kilograms? Juanita used the steps shown below to correctly solve an equation. A step is missing.-3(c-6) + 4c = 5(2c + 9)?c+ 18 = 10c +45c-27 = 10 c-27= 9c-3 = cO-3c-6 + 4c = 10c + 9; associative property of additionO-3c+ 18+ 4c = 10c + 45; associative property of additionO-3c-6 + 4c = 10c + 9; distributive propertyO-3c + 18+ 4c = 10c +45; distributive property Given that f ( x ) = 3 x 6 f ( x ) = 3 x - 6 and g ( x ) = 2 x 2 g ( x ) = 2 - x 2 , calculate Number 6. Find the missing measure and round to nearest 10th Choose the word or phrase that uses the correct verb tense in the underlined section of the sentence belowWhat I needed( is )solitude to sort out the confusion and uneasiness.O had beenO wasO werecorrect as is Charmaine has b decks of cards. Each deck has 52 cards in it. Using b, write an expression for the total number of cards Charmaine has you are choosing between two different cell phone plans. the first plan charges a rate of 23 cents per minute. the second plan charges a monthly fee of $34.95 plus 10 cents per minute. 2. Which describes the alveoli?O They have very thick walls to contain the air that enters the lungs.They work with the villi to exchange oxygen and carbon dioxide.They provide a large amount of surface area for gas exchange.They are structures responsible for the production of our voice. p and q are both prime numbers. They are each less than 22 Give an example where p + q is odd but not prime. You must only write the two numbers in the answer box. Type here to search O RI e Total ma Heres the question to solve. Just do the question that has the chart Explain the role of the glands in the brain What is the volume, in cubic inches, of a rectangular prism with a height of 19in, a width of 8in, and a length of 17in? Which of the following statements is INCORRECT?a) The Mayan civilization arose in the highland areas of Mexico.b) The Aztec civilization was founded after the Maya Civilization.c) The ceremonial center of the Aztec civilization, named Tenochtitlan, was located in the Valley of Mexico.d) Maya languages are still used today in parts of Mexico.e) The Aztecs were conquered by the Spanish. How does senator obamas use of the metaphor ""beacon of freedom"" reflect the purpose of inspiring the audience to take action in the upcoming election? select all that apply. 1) Choose whether thyroid or thymus applies to each of the following:WhiteDark redShaped like a VShaped like a small bean Need answer as soon as possible, emergency, please be 100% sure, thank you Help need please!!!!!!!factorize 36a-12b-60c